If you find any mistakes, please make a comment! Thank you.

Chapter 3 Exercise E


Exercises 1,2 and 4. For Problem 2, please also see Carson Rogers’s comment.


4. Solution: For any $f\in \ca L(V_1\times \cdots\times V_m,W)$ and given $i\in \{1,\cdots,m\}$, define $f_i:V_i\to W$ by the rule:\[f_i(v_i)=f(0,\cdots,0,v_i,0,\cdots,0),\]where $v_i$ sits in the $i$-th slot. One can check that $f_i\in\ca L(V_i,W)$.

Define $\vp :\ca L(V_1\times \cdots\times V_m,W)\to \ca L(V_1,W)\times \cdots \times \ca L(V_m,W)$ by\[\vp(f)=(f_1,\cdots,f_m),\] where $f_1$, $\cdots$, $f_m$ are defined in the previous paragraph. Now we are going to check that $\vp$ is an isomorphism between $\ca L(V_1\times \cdots\times V_m,W)$ and $\ca L(V_1,W)\times \cdots \times \ca L(V_m,W)$ by construct an inverse map.

Define $\psi :\ca L(V_1,W)\times \cdots \times \ca L(V_m,W)\to \ca L(V_1\times \cdots\times V_m,W)$ by\[\psi(f_1,\cdots,f_m)(v_1,\cdots,v_m)=f_1(v_1)+\cdots+f_m(v_m).\]

Please check that $\psi$ and $\vp$ are linear. It is also not hard to see that $\psi\circ \vp=Id$, and $\vp\circ \psi=Id$. Hence we are done.


6. (By YiweiJiang2015) To prove that $V^n$ and $\mathcal{L}(\mathbf{F}^n, V)$ are isomorphic vector spaces, we need to establish a bijective linear map between them.

Let’s define the map $\Phi: V^n \rightarrow \mathcal{L}(\mathbf{F}^n, V)$ as follows:

For any vector $\mathbf{v} = (v_1, v_2, \ldots, v_n) \in V^n$, we define $\Phi(\mathbf{v}): \mathbf{F}^n \rightarrow V$ as
\[
\Phi(\mathbf{v})(\mathbf{f}) = v_1 \mathbf{f}_1 + v_2 \mathbf{f}_2 + \ldots + v_n \mathbf{f}_n,
\]
where $\mathbf{f} = (\mathbf{f}_1, \mathbf{f}_2, \ldots, \mathbf{f}_n) \in \mathbf{F}^n, \mathbf{f}_j \in \mathbf{F}$.

Now, let’s show that $\Phi$ is a well-defined linear map and establish its properties:

Linearity:

Addition preservation: For any $\mathbf{v}_1, \mathbf{v}_2 \in V^n$, and $\mathbf{f} \in \mathbf{F}^n$, we have:
\begin{align*}
\Phi(\mathbf{v}_1 + \mathbf{v}_2)(\mathbf{f}) &= (v_{1,1} + v_{2,1})\mathbf{f}_1 + (v_{1,2} + v_{2,2})\mathbf{f}_2 + \ldots + (v_{1,n} + v_{2,n})\mathbf{f}_n \\
&= (v_{1,1}\mathbf{f}_1 + v_{1,2}\mathbf{f}_2 + \ldots + v_{1,n}\mathbf{f}_n) + (v_{2,1}\mathbf{f}_1 + v_{2,2}\mathbf{f}_2 + \ldots + v_{2,n}\mathbf{f}_n) \\
&= \Phi(\mathbf{v}_1)(\mathbf{f}) + \Phi(\mathbf{v}_2)(\mathbf{f})
\end{align*}
Thus, addition is preserved under $\Phi$.

Scalar multiplication preservation: For any scalar $c \in \mathbf{F}$, vector $\mathbf{v} \in V^n$, and $\mathbf{f} \in \mathbf{F}^n$, we have:
\begin{align*}
\Phi(c\mathbf{v})(\mathbf{f}) &= (cv_{1,1})\mathbf{f}_1 + (cv_{1,2})\mathbf{f}_2 + \ldots + (cv_{1,n})\mathbf{f}_n \\
&= c(v_{1,1}\mathbf{f}_1 + v_{1,2}\mathbf{f}_2 + \ldots + v_{1,n}\mathbf{f}_n) \\
&= c\Phi(\mathbf{v})(\mathbf{f})
\end{align*}
Thus, scalar multiplication is preserved under $\Phi$.

Injectivity: To show that $\Phi$ is injective, we need to prove that if $\Phi(\mathbf{v}_1) = \Phi(\mathbf{v}_2)$, then $\mathbf{v}_1 = \mathbf{v}_2$.

Suppose $\Phi(\mathbf{v}_1) = \Phi(\mathbf{v}_2)$ for some $\mathbf{v}_1, \mathbf{v}_2 \in V^n$. Let $\mathbf{f} = (\mathbf{f}_1, \mathbf{f}_2, \ldots, \mathbf{f}_n) \in \mathbf{F}^n$. Then we have:
\begin{align*}
\Phi(\mathbf{v}_1)(\mathbf{f}) &= \Phi(\mathbf{v}_2)(\mathbf{f}) \\
v_{1,1}\mathbf{f}_1 + v_{1,2}\mathbf{f}_2 + \ldots + v_{1,n}\mathbf{f}_n &= v_{2,1}\mathbf{f}_1 + v_{2,2}\mathbf{f}_2 + \ldots + v_{2,n}\mathbf{f}_n
\end{align*}

Since this equation holds for any $\mathbf{f} \in \mathbf{F}^n$, it must hold for the standard basis vectors of $\mathbf{F}^n$. Thus, we have:
\begin{align*}
v_{1,1} &= v_{2,1} \\
v_{1,2} &= v_{2,2} \\
&\vdots \\
v_{1,n} &= v_{2,n}
\end{align*}

Therefore, we conclude that $\mathbf{v}_1 = \mathbf{v}_2$, and hence, $\Phi$ is injective.

Surjectivity: To show that $\Phi$ is surjective, we need to prove that for any $\mathcal{T} \in \mathcal{L}(\mathbf{F}^n, V)$, there exists $\mathbf{v} \in V^n$ such that $\Phi(\mathbf{v}) = \mathcal{T}$.

Let $\mathcal{T} \in \mathcal{L}(\mathbf{F}^n, V)$ be an arbitrary linear map. We define $\mathbf{v} = (\mathcal{T}(\mathbf{e}_1), \mathcal{T}(\mathbf{e}_2), \ldots, \mathcal{T}(\mathbf{e}_n)) \in V^n$, where $\mathbf{e}_i$ is the $i$-th standard basis vector of $\mathbf{F}^n$. Now, let’s show that $\Phi(\mathbf{v}) = \mathcal{T}$.

For any $\mathbf{f} = (\mathbf{f}_1, \mathbf{f}_2, \ldots, \mathbf{f}_n) \in \mathbf{F}^n$, we have:
\begin{align*}
\Phi(\mathbf{v})(\mathbf{f}) &= \Phi(\mathcal{T}(\mathbf{e}_1), \mathcal{T}(\mathbf{e}_2), \ldots, \mathcal{T}(\mathbf{e}_n))(\mathbf{f}) \\
&= \mathcal{T}(\mathbf{e}_1)\mathbf{f}_1 + \mathcal{T}(\mathbf{e}_2)\mathbf{f}_2 + \ldots + \mathcal{T}(\mathbf{e}_n)\mathbf{f}_n \\
&= \mathcal{T}(\mathbf{e}_1\mathbf{f}_1 + \mathbf{e}_2\mathbf{f}_2 + \ldots + \mathbf{e}_n\mathbf{f}_n) \\
&= \mathcal{T}(\mathbf{f})
\end{align*}
The third equality holds because $\mathcal{T}$ is a linear map.
Hence, we have $\Phi(\mathbf{v}) = \mathcal{T}$, which implies that $\Phi$ is surjective.

Since $\Phi$ is injective and surjective, it is a bijective linear map between $V^n$ and $\mathcal{L}(\mathbf{F}^n, V)$. Therefore, $V^n$ and $\mathcal{L}(\mathbf{F}^n, V)$ are isomorphic vector spaces.


7. Solution: Note that $v+U=x+W$, hence $v=x+w_1$, where $w_1\in W$. It follows that $v-x\in W$. Hence for any $u\in U$, we have \[ v+u=x+w_2 \]for some $w_2\in W$ since $v+U=x+W$. Hence we conclude that \[u=(x-v)+w_2=-w_1+w_2\in W,\] it follows that $U\subset W$ for $u$ is chosen arbitrarily. Similarly, we deduce that $W\subset U$. Thus $U=W$.


8. Solution: If $A$ is an affine subset of $V$, then there exist a vector $a\in V$ and a subspace $U$ of $V$ such that $A=a+U$. Then any $v,w\in A$ can be written as $v=a+u_1$ and $w=a+u_2$ for some $u_1,u_2\in U$. Hence \[ \lambda v+(1-\lambda)w=a+[\lambda u_1+(1-\lambda)u_2]\in a+U=A. \] Conversely, since $A$ is nonempty, let $a\in A$. We will show that \[A-a=\{x-a:x\in A\}\]is a subspace of $V$. For $x-a\in A-a$ and $\lambda\in \mb F$ where $x\in A$, then \[ \lambda x+(1-\lambda)a\in A\Longrightarrow \lambda (x-a)=\lambda x+(1-\lambda)a-a\in A-a. \]This implies $A-a$ is closed under scalar multiplication. For $x-a$ and $y-a\in A-a$, where $x,y\in A$. We have \[ \frac{x}{2}+\frac{y}{2}-a\in A-a. \]Note that $A-a$ is closed under scalar multiplication, it follows that \[ (x-a)+(y-a)=2\left(\frac{x}{2}+\frac{y}{2}-a\right)\in A-a. \]That is $A-a$ is closed under addition. Hence $A-a$ is a subspace of $V$. Note that $A=a+(A-a)$, it follows that $A$ is an affine subset of $V$.


9. Solution: Suppose $A_1\cap A_2\ne \varnothing$, then for any $x,y\in A_1\cap A_2$ and $\lambda\in \mb F$, we have \[ \lambda x+(1-\lambda) y\in A_1 \]and \[ \lambda x+(1-\lambda) y\in A_2 \]by Problem 8 since $A_1$ and $A_2$ are affine subsets of $V$. Hence \[ \lambda x+(1-\lambda) y\in A_1\cap A_2. \]Again by Problem 8, we deduce that $A_1\cap A_2$ is an affine subset of $V$.


10. Solution: It is the same as Problem 9.


11. Solution: (a) For $v=\lambda_1 v_1+\cdots+\lambda_m v_m\in A$ and $w=\eta_1 v_1+\cdots+\eta_m v_m\in A$,where $\lambda_1,\cdots,\lambda_m\in\mb F$, $\lambda_1+\cdots+\lambda_m=1$ and $\eta_1,\cdots,\eta_m\in\mb F$, $\eta_1+\cdots+\eta_m=1$. For any $\lambda\in\mb F$, we have \begin{align*} \lambda v+(1-\lambda) w=\sum_{i=1}^m(\lambda\lambda_i+(1-\lambda)\eta_i)v_i. \end{align*} Note that \[ \sum_{i=1}^m(\lambda\lambda_i+(1-\lambda)\eta_i)=\lambda\sum_{i=1}^m\lambda_i+(1-\lambda)\sum_{i=1}^n\eta_i=\lambda+(1-\lambda)=1, \]we deduce that $\lambda v+(1-\lambda) w\in A$. Hence $A$ is an affine subset of $V$ by Problem 8.

(b) We will use induction to show that for any affine subset $W$ of $V$ that contains $v_1$, $\cdots$, $v_m$$k\le m$, then if $\lambda_1+\cdots+\lambda_k=1$, we have \[ \sum_{j=1}^k\lambda_jv_j\in W. \]It is obvious for $k=1$ and $k=2$ by Problem 8. Suppose this is true for $k$, then we will show it for $k+1$ ($k+1\le m$). Assume $\lambda_1+\cdots+\lambda_{k+1}=1$. If $\lambda_{k+1}=1$, then \[\sum_{j=1}^{k+1}\lambda_jv_j=v_{k+1}\in W.\]If $\lambda_{k+1}\ne 1$, then \[ \frac{1}{1-\lambda_{k+1}}(\lambda_1+\cdots+\lambda_k)=1. \]Hence by assumption, we deduce that \[ \frac{1}{1-\lambda_{k+1}}(\lambda_1v_1+\cdots+\lambda_kv_k)\in W. \]By Problem 8, we have \[ (1-a_{k+1})\left(\frac{1}{1-\lambda_{k+1}}(\lambda_1v_1+\cdots+\lambda_kv_k)\right)+a_{k+1}v_{k+1}\in W, \]namely \[ \lambda_1v_1+\cdots+\lambda_{k+1}v_{k+1}\in W. \]By mathematical induction, we conclude if $\lambda_1,\cdots,\lambda_m\in\mb F$, $\lambda_1+\cdots+\lambda_m=1$, then \[\lambda_1 v_1+\cdots+\lambda_m v_m\in W,\]thus $A\subset W$.

(c) Note that if $\lambda_1+\cdots+\lambda_m=1$, we have \[ \lambda_1 v_1+\cdots+\lambda_m v_m=v_1+\lambda_2(v_2-v_1)+\cdots+\lambda_m(v_m-v_1). \]Hence $A\subset v_1+\m{span}(v_2-v_1,\cdots,v_m-v_1)$. Similarly, for any\[v\in v_1+\m{span}(v_2-v_1,\cdots,v_m-v_1),\]$v$ can be written as \[ v_1+\sum_{i=2}^m \lambda_i(v_i-v_1)=(1-\lambda_2-\cdots-\lambda_m)v_1+\sum_{i=2}^m \lambda_iv_i \]for some $\lambda_2$, $\cdots$, $\lambda_m\in\mb F$. Note that \[ (1-\lambda_2-\cdots-\lambda_m)+\sum_{i=2}^m \lambda_i=1, \]we deduce that $v_1+\m{span}(v_2-v_1,\cdots,v_m-v_1)\subset A$. Hence\[A=v_1+\m{span}(v_2-v_1,\cdots,v_m-v_1).\]Let $v=v_1$ and $U=\m{span}(v_2-v_1,\cdots,v_m-v_1)$, then $\dim U\le m-1$.


12. Solution: Since $V/U$ is finite-dimensional, we can suppose $v_1+U$, $\cdots$, $v_n+U$ is a basis of $V/U$. Then for any $v\in V$, there exist a unique list of $\lambda_1$, $\cdots$, $\lambda_n\in\mb F$ such that \[ v+U=\sum_{i=1}^n\lambda_i(v_i+U). \]Then\[v-\sum_{i=1}^n\lambda_iv_i\in U.\]Define $\vp:V\to U\times V/U$ by \[\vp(v)=\left(v-\sum_{i=1}^n\lambda_iv_i,\sum_{i=1}^n\lambda_i(v_i+U)\right).\]We first check that $\vp$ is linear. For any $v,w\in V$, we \[ v+U=\sum_{i=1}^n\lambda_i(v_i+U) \]and \[ w+U=\sum_{i=1}^n\eta_i(v_i+U). \]for some $\lambda_1$, $\cdots$, $\lambda_n\in\mb F$ and $\eta_1$, $\cdots$, $\eta_n\in\mb F$. Then for any $\lambda \in \mb F$, we have \[ (v+\lambda w)+U=(v+U)+\lambda(w+U)=\sum_{i=1}^n(\lambda_i+\lambda\eta_i)(v_i+U). \]Hence \[ \vp(v+\lambda w)=\left(v+\lambda w-\sum_{i=1}^n(\lambda_i+\lambda\eta_i)v_i,\sum_{i=1}^n(\lambda_i+\lambda\eta_i)(v_i+U)\right), \]and \[ \vp(v)=\left(v-\sum_{i=1}^n\lambda_iv_i,\sum_{i=1}^n\lambda_i(v_i+U)\right), \]and \[ \vp(w)=\left(w-\sum_{i=1}^n\eta_iv_i,\sum_{i=1}^n\eta_i(v_i+U)\right). \]Therefore we have (check it) \begin{align*} \vp(v)+\lambda\vp( w)=\vp(v+\lambda w), \end{align*} namely $\vp$ is linear.

Injectivity: if $\vp(v)=0$, then $\lambda_1=\cdots=\lambda_n=0$(defined as above) and \[0=v-\sum_{i=1}^n\lambda_iv_i=v.\] Surjectivity: For $\left(u,\sum_{i=1}^n\xi(v_i+U)\right)\in U\times V/U$, it is easy to see \[ \vp\left(u+\sum_{i=1}^n\xi v_i\right)=\left(u,\sum_{i=1}^n\xi(v_i+U)\right). \]Hence $\vp$ is an isomorphism, namely $V$ is isomorphic to $U\times V/U$.


13. Solution: For any $v\in V$, since $v_1+U$, $\cdots$, $v_m+U$ is a basis of $V/U$, there exist $\lambda_1$, $\cdots$, $\lambda_m\in\mb F$ such that \[ v+U=\sum_{i=1}^m \lambda_i(v_i+U). \]This implies \[ v-\sum_{i=1}^m\lambda_iv_i\in U. \]Note that $u_1$, $\cdots$, $u_n$ is a basis of $U$, it follows that \[ v-\sum_{i=1}^m\lambda_iv_i=\sum_{j=1}^n\eta_ju_j \]for some $\eta_1$, $\cdots$, $\eta_n\in \mb F$. Hence \[ v=\sum_{i=1}^m\lambda_iv_i+\sum_{j=1}^n\eta_ju_j, \]this implies $V=\m{span}(v_1,\cdots,v_m,u_1,\cdots,u_n)$ since $v$ is chosen arbitrarily. Hence it suffices to show that $v_1$, $\cdots$, $v_m$, $u_1$, $\cdots$, $u_n$ is linearly independent. Suppose for some $\lambda_1$, $\cdots$, $\lambda_m\in\mb F$ and $\eta_1$, $\cdots$, $\eta_n\in \mb F$, we have \[ \sum_{i=1}^m\lambda_iv_i+\sum_{j=1}^n\eta_ju_j=0. \]Then \[ \sum_{i=1}^m\lambda_i(v_i+U)=0, \]hence $\lambda_1=\cdots=\lambda_m=0$ since $v_1+U$, $\cdots$, $v_m+U$ is a basis of $V/U$. It follows that \[\sum_{j=1}^n\eta_ju_j=0.\]Note that $u_1$, $\cdots$, $u_n$ is a basis of $U$, we obtain that $\eta_1=\cdots=\eta_n=0$. Hence $v_1$, $\cdots$, $v_m$, $u_1$, $\cdots$, $u_n$ is linearly independent.


14. Solution:

a) Simply note that multiplying a vector $u \in U$ by a scalar doesn’t increase the number of it’s non-zero entries, and that given vectors $u_1 \in U$ with $M_1$ non-zero entries and $u_2 \in U$ with $M_2$ non-zero entries, the sum $u_1 + u_2$ has at most $M_1 + M_2$ non-zero entries.

b) Throughout this exercise, given a vector $v \in \boldsymbol{F}^{\infty}$, we’ll denote by $v(p)$ the number on the $p$-th entry of $v$.

Consider the list of vectors $e_m$ for $m=1,2,\cdots$ in $\boldsymbol{F}^{\infty}$ defined by the rule:

$$
e_m(p) = \cases{1, & if $(p-1) \mod m = 0$ \cr
0, & otherwise }
$$

For example,

$$e_1 = (1,1,1,1,1,1,…)$$

$$e_2 = (1,0,1,0,1,0,…)$$

$$e_3 = (1,0,0,1,0,0,…)$$ $$…$$

We want to show that this list gives a corresponding linear independent list on $\boldsymbol{F}^{\infty}/U$. Let $k_1,k_2,…k_m \in \boldsymbol{F}$ such that: $$k_1(e_1 + U) + k_2(e_2 + U) + … + k_m(e_m + U) = 0 + U$$

Note that $k_1(e_1 + U) + … + k_m(e_m + U) = (k_1 e_1+ … + k_m e_m) + U$, so the above equality implies that $k_1 e_1+ … + k_m e_m = u$ for some $u \in U$. Since $u$ has a finite number of non-zero elements, it must contain only zeroes from some position onward. Let $L$ be the last position of $u$ that might contain a non-zero entry, that is, $u(i) = 0$ for any $i > L$. Now, let $p = t \cdot m! + 1$, where we choose $t$ so $p \geq L $. Our choice of $p$ imples that $e_1(p) = e_2(p) = … = e_m(p) = 1$ and that $u(q) = 0$ for any $q > p$.

Now we proceed to show that $k_1 = k_2 = … = k_m = 0$, proving the desired linear independence. Note that $e_1(p+1) = 1$ while $e_2(p+1) = e_3(p+1) = … = e_m(p+1) = 0$, which means $u(p+1) = k_1 e_1(p+1) + … + k_m e_m(p+1) = k_1$. Since $u(p+1) = 0$, this implies $k_1 = 0$. Similarly, we have $e_2(p+2) = 1$ while $e_3(p+2) = e_4(p+2) = … = e_m(p+2) = 0$, and since $k_1 = 0$, $u(p+2) = k_2$, so $k_2 = 0$. Following the induction, it’s possible to show the desired result that $k_1 = … = k_m = 0$.

We constructed a list of vectors in $\boldsymbol{F}^{\infty}/U$ of arbitrary size that is linearly independent (namely, the list $e_1 + U,…,e_m + U$). This shows that $\boldsymbol{F}^{\infty}/U$ is infinite dimensioned, since no finite l.i. list can span the whole vector space.


15. Solution: Note that $\varphi:V\to \mb F$ and $\varphi\ne 0$, there exists $v\in V$ such that $\varphi(v)\ne 0$. Then for any $a\in\mb F$, we have\[\vp\Big(\frac{av}{\varphi(v)}\Big)=\frac{a}{\varphi(v)}\vp(v)=a.\]Therefore $\mathrm{range}\,\vp=\mb F$.

By 3.91(d), we have $V/\mathrm{null}\,\vp\cong \mb F$. Hence $$\dim V/\mathrm{null}\,\vp=\dim \mb F=1.$$


16. Solution: Since $\dim V/U=1$, there exists a vector $v\notin U$ such that $v+U$ is a basis of $V/U$. Define $\theta: V/U\to \mb F$ by \[\theta(kv+U)=k,\]then $\theta\in\ca L(V/U,\mb F)$.

Let $\pi$ be the quotient map $\pi :V\to V/U$. Let $\vp=\theta\circ\pi$, then $\vp\in\ca L(V,\mb F)$. Now we check that $\mathrm{null}\,\vp=U$.

Let $w\in V$ such that $\vp(w)=0$, i.e. $\theta(\pi(w))=0$. By the definition of $\theta$, we have $$w+U=0v+U.$$ Hence $w\in U$, which implies that $\mathrm{null}\,\vp\subset U$.

On the other hand, let $u\in U$, then $\pi(u)=u+U=0+U$. Hence $\theta\circ \pi(u)=0$, which implies that $U\subset \mathrm{null}\,\vp$.

Therefore $\mathrm{null}\,\vp=U$. (Please note that such $\vp$ is not unique.)


17. Solution: Since $V/U$ is finite-dimensional, let $n=\dim (V/U)$, then there exist $w_1,\cdots,w_n\in V$ such that $w_1+U$, $\cdots$, $w_n+U$ form a basis of $V/U$. It is easy to see that $w_1,\cdots,w_n$ is linearly independent otherwise $w_1+U$, $\cdots$, $w_n+U$ would be linearly dependent. Consider the subspace $W$ of $V$ spanned by $w_1,\cdots,w_n$, then we have $\dim W=n=\dim(V/U)$.

Now we are going to show that $V=U\oplus W$. For any $v\in V$, since $w_1+U$, $\cdots$, $w_n+U$ is a basis of $V/U$, there exist $k_1,\cdots,k_n\in\mb F$ such that $$v+U=\sum_{i=1}^n k_i(w_i+U).$$This implies that $$v-\sum_{i=1}^n k_iw_i\in U.$$Therefore consider \[v=\Big(v-\sum_{i=1}^n k_iw_i\Big)+\sum_{i=1}^n k_iw_i,\]it tells us that $v\in U+W$. Since $v$ is chosen arbitrarily, we conclude that $V\subset U+W$. To show that $V=U\oplus W$, it suffices to show that $U\cap W=0$.

Suppose $v\in U\cap W$, then there exist $k_1,\cdots,k_n\in\mb F$ such that $$v=\sum_{i=1}^n k_iw_i.$$Hence we have $0=\pi(v)=\sum_{i=1}^n k_i(w_i+U)$, where $\pi$ is the canonical quotient map from $V$ onto $V/U$. However, note that $w_1+U$, $\cdots$, $w_n+U$ is a basis of $V/U$ we have $k_1=\cdots=k_n=0$. Thus$$v=\sum_{i=1}^n k_iw_i=0.$$This shows that $U\cap W=0$.


18. Solution: Suppose there exists $S\in\ca L(V/U,W)$ such that $T=S\circ\pi$. Since for any $u\in U$ we have $\pi u=0$, hence $$Tu=(S\circ \pi )u=S(\pi u)=S0=0.$$Hence $u\in \mathrm{null}\, T$. Note that this is true for any $u\in U$, we conclude that $U\subset \mathrm{null}\, T$.

Conversely, suppose $U\subset \mathrm{null}\, T$. Define a map $S: V/U\to W$ by $S(v+U)=Tv$. We have to check that this map is well-defined (since there may exist different $v_1,v_2\in V$ such that $v_1+U=v_2+U$ and we need to check that $Tv_1=Tv_2$).

For $v_1,v_2\in V$ such that $v_1+U=v_2+U$, we have $v_1-v_2\in U$. Note that $U\subset \mathrm{null}\, T$, it follows that $T(v_1-v_2)=0$, hence we have $Tv_1=Tv_2$. This implies that $S$ is well-defined.

Note that $T\in \ca L(V,W)$, it is easy to see that $S\in \ca L(V/U,W)$. By definition, we have $$S\circ \pi (v)=S(v+U)=Tv$$ for any $v\in V$. This implies that $T=S\circ \pi$.


19. Solution: For any given finite set $A$, denote by $|A|$ the number of elements in $A$. Let $S_i$, $i=1,\cdots,n$, be finite sets. The union $\bigcup_{i=1}^n S_i$ is a disjoint union if and only if $$|S_1|+\cdots+|S_n|=\Big|\bigcup_{i=1}^n S_i\Big|.$$


20. Solution: (a) To show that $\Gamma$ is linear, we need to show that for any $S_1,S_2\in\ca L(V/U,W)$ and $k_1,k_2\in\mb F$ we have $$\Gamma(k_1S_1+k_2S_2)=k_1\Gamma(S_1)+k_2\Gamma(S_2).$$This follows directly since \begin{align*}\Gamma(k_1S_1+k_2S_2)=&(k_1S_1+k_2S_2)\circ \pi\\=& k_1S_1\circ \pi+k_2S_2\circ \pi\\=& k_1\Gamma(S_1)+k_2\Gamma(S_2).\end{align*}(b) To show that $\Gamma$ is injective, one has to show that if $\Gamma(S)=0$ then $S=0$. If $\Gamma(S)=0$, it implies that $S\circ \pi=0$. Hence $(S\circ \pi) v=0$ for all $v\in V$. Thus $S(\pi(v))=S(v+U)=0$ for all $v\in V$. We conclude that $S=0$, which shows $\Gamma$ is injective.

(c) By Problem 18, we have\begin{align*}T\in \mathrm{range}\, \Gamma\iff & \text{there exists }S\in \ca L(V/U,W)\text{ s.t. }T=S\circ \pi\\ \iff & U\subset \mathrm{null}\, T\quad \text{ by Problem 18}\\ \iff &T\in\ca L(V,W): \, Tu=0 \text{ for every } u\in U.\end{align*}This completes the proof.


Linearity

This website is supposed to help you study Linear Algebras. Please only read these solutions after thinking about the problems carefully. Do not just copy these solutions.
Close Menu